USAMO 2017 Solutions by Evan Chen
USAMO 2017 Solutions by Evan Chen
USAMO 2017 Solutions by Evan Chen
Evan Chen《陳誼廷》
2 June 2023
Contents
0 Problems 2
1 Solutions to Day 1 3
1.1 USAMO 2017/1, proposed by Gregory Galperin . . . . . . . . . . . . . . . 3
1.2 USAMO 2017/2, proposed by Maria Monks . . . . . . . . . . . . . . . . . 4
1.3 USAMO 2017/3, proposed by Evan Chen . . . . . . . . . . . . . . . . . . 7
2 Solutions to Day 2 9
2.1 USAMO 2017/4, proposed by Maria Monks . . . . . . . . . . . . . . . . . 9
2.2 USAMO 2017/5, proposed by Ricky Liu . . . . . . . . . . . . . . . . . . . 12
2.3 USAMO 2017/6, proposed by Titu Andreescu . . . . . . . . . . . . . . . . 14
1
USAMO 2017 Solution Notes web.evanchen.cc, updated 2 June 2023
§0 Problems
1. Prove that there exist infinitely many pairs of relatively prime positive integers
a, b > 1 for which a + b divides ab + ba .
3. Let ABC be a scalene triangle with circumcircle Ω and incenter I. Ray AI meets
BC at D and Ω again at M ; the circle with diameter DM cuts Ω again at K.
Lines M K and BC meet at S, and N is the midpoint of IS. The circumcircles of
4KID and 4M AN intersect at points L1 and L2 . Prove that Ω passes through
the midpoint of either IL1 or IL2 .
5. Find all real numbers c > 0 such that there exists a labeling of the lattice points in
Z2 with positive integers for which:
• only finitely many distinct labels occur, and
• for each label i, the distance between any two points labeled i is at least ci .
2
USAMO 2017 Solution Notes web.evanchen.cc, updated 2 June 2023
§1 Solutions to Day 1
§1.1 USAMO 2017/1, proposed by Gregory Galperin
Available online at https://aops.com/community/p8108366.
Problem statement
Prove that there exist infinitely many pairs of relatively prime positive integers
a, b > 1 for which a + b divides ab + ba .
x+d x−d
a= , b= .
2 2
To see this works, first check that b is odd and a is even. Let d = a − b be odd. Then:
a + b | ab + ba ⇐⇒ (−b)b + ba ≡ 0 (mod a + b)
a−b
⇐⇒ b ≡1 (mod a + b)
⇐⇒ bd ≡ 1 (mod d + 2b)
⇐⇒ (−2) ≡ dd d
(mod d + 2b)
⇐⇒ d + 2b | dd + 2d .
dd + 2d dd + 2d
1
d + 2b = =⇒ b = −d
d+2 2 d+2
3
USAMO 2017 Solution Notes web.evanchen.cc, updated 2 June 2023
Problem statement
• ai ≥ wi > wj ,
• wj > ai ≥ wi , or
• wi > w j > a i .
Show that, for any two sequences of integers A = (a1 , . . . , an ) and B = (b1 , . . . , bn ),
and for any positive integer k, the number of permutations of m1 , . . . , mn having
exactly k A-inversions is equal to the number of permutations of m1 , . . . , mn having
exactly k B-inversions.
The following solution was posted by Michael Ren, and I think it is the most natural one
(since it captures all the combinatorial ideas using a q-generating function that is easier
to think about, and thus makes the problem essentially a long computation).
Denote by M our multiset of n positive integers. Define an inversion of a permutation to
be pair i < j with wi < wj (which is a (0, . . . , 0)-inversion in the problem statement); this
is the usual definition (see https://en.wikipedia.org/wiki/Inversion_(discrete_
mathematics)). So we want to show the number of A-inversions is equal to the number
of usual inversions. In what follows we count permutations on M with multiplicity: so
M = {1, 1, 2} still has 3! = 6 permutations.
We are going to do what is essentially recursion, but using generating functions in
a variable q to do our book-keeping. (Motivation: there’s no good closed form for the
number of inversions, but there’s a great generating function known — which is even
better for us, since we’re only trying to show two numbers are equal!) First, we prove
two claims.
Claim — For any positive integer n, the generating function for the number of
permutations of (1, 2, . . . , n) with exactly k inversions is
n!q := 1 · (1 + q) · (1 + q + q 2 ) · . . . (1 + q + · · · + q n−1 ).
Here we mean that the coefficient of q s above gives the number of permutations with
exactly s inversions.
Proof. This is an induction on n, with n = 1 being trivial. Suppose we choose the first
element to be i, with 1 ≤ i ≤ n. Then there will always be exactly i − 1 inversions
using the first element, so this contributes q i · (n − 1)!q . Summing 1 ≤ i ≤ n gives the
result.
Unfortunately, the main difficulty of the problem is that there are repeated elements,
which makes our notation much more horrific.
4
USAMO 2017 Solution Notes web.evanchen.cc, updated 2 June 2023
Let us define the following. We take our given multiset M of n positive integers, we
suppose the distinct numbers are θ1 < θ2 < · · · < θm . We let ei be the number of times
θi appears. Therefore the multiplicities ei should have sums
e1 + · · · + em = n
q number inversions of σ
X
F (e1 , . . . , em ) =
permutations σ
be the associated generating function for the number of inversions. For example, the first
claim we proved says that F (1, . . . , 1) = n!q .
Proof. First suppose we perturb all the elements slightly, so that they are no longer equal.
Then the generating function would just be n!q .
Then, we undo the perturbations for each group, one at a time, and claim that we get
the above ei !q factor each time. Indeed, put the permutations into classes of e1 ! each
where permutations in the same classes differ only in the order of the perturbed θ1 ’s
(with the other n − e1 elements being fixed). Then there is a factor of e1 !q from each class,
owing to the slightly perturbed inversions we added within each class. So we remove that
factor and add e1 ! · q 0 instead. This accounts for the first term of the product.
Repeating this now with each term of the product implies the claim.
Thus we have the formula for the number of inversions in general. We wish to show
this also equals the generating function the number of A-inversions, for any fixed choice
of A. This will be an induction by n, with the base case being immediate.
For the inductive step, fix A, and assume the first element satisfies θk ≤ a1 < θk+1 (so
0 ≤ k ≤ m; we for convenience set θ0 = −∞ and θm = +∞). We count the permutations
based on what the first element θi of the permutation is. Then:
• Consider permutations starting with θi ∈ {θ1 , . . . , θk }. Then the number of inver-
sions which will use this first term is (e1 + · · · + ei−1 ) + (ek+1 + · · · + em ). Also, there
are ei ways to pick which θi gets used as the first term. So we get a contribution of
• Now suppose θi ∈ {θk+1 , . . . , θm }. Then the number of inversions which will use
this first term is ek+1 + · · · + ei−1 . Thus by a similar argument the contribution is
5
USAMO 2017 Solution Notes web.evanchen.cc, updated 2 June 2023
m
X
+ q ek+1 +···+ei−1 · ei · F (e1 , . . . , ei − 1, . . . , em )
i=k+1
= F (e1 , . . . , em ).
ei · F (e1 , . . . , ei − 1, . . . , em ) 1 + · · · + q ei −1 1 − q ei
= =
F (e1 , . . . , em ) 1 + q + · · · + q n−1 1 − qn
which is clear, since the left summand telescopes to q ek+1 +···+em − q n and the right
summand telescopes to 1 − q ek+1 +···+em .
Remark. Technically, we could have skipped straight to the induction, without proving the
first two claims. However I think the solution reads more naturally this way.
6
USAMO 2017 Solution Notes web.evanchen.cc, updated 2 June 2023
Problem statement
Let ABC be a scalene triangle with circumcircle Ω and incenter I. Ray AI meets
BC at D and Ω again at M ; the circle with diameter DM cuts Ω again at K.
Lines M K and BC meet at S, and N is the midpoint of IS. The circumcircles of
4KID and 4M AN intersect at points L1 and L2 . Prove that Ω passes through
the midpoint of either IL1 or IL2 .
Let W be the midpoint of BC, let X be the point on Ω opposite M . Observe that KD
passes through X, and thus lines BC, M K, XA concur at the orthocenter of 4DM X,
which we call S. Denote by IA the A-excenter of ABC.
Next, let E be the foot of the altitude from I to XIA ; observe that E lies on the
circle centered at M through I, B, C, IA . Then, S is the radical center of Ω and the
circles with diameter IX and IIA ; hence line SI passes through E; accordingly I is the
orthocenter of 4XSIA ; denote by L the foot from X to SIA .
I
N
S B D W C
T
K
L M
IA
We claim that this L lies on both the circumcircle of 4KID and 4M AN . It lies on
the circumcircle of 4M AN since this circle is the nine-point circle of 4XSIA . Also,
XD · XK = XW · XM = XA · XS = XI · XL, so KDIL are concyclic.
All that remains to show is that the midpoint T of IL lies on Ω. But this follows from
the fact that T M k LIA =⇒ ∠M T X = 90◦ , thus the problem is solved.
Remark. Some additional facts about this picture: the point T is the contact point of the
A-mixtilinear incircle (since it is collinear with X and I), while the point K is such that
AK is an A-symmedian (since KD and AD bisect ∠A and ∠K, say).
7
USAMO 2017 Solution Notes web.evanchen.cc, updated 2 June 2023
IA
M
B
L K
T D
C
N I
S IB A X IC
Remark. In fact, the point L is the Miquel point of cyclic quadrilateral IB IC BC (inscribed
in the circle with diameter IB IC ). This implies many of the properties that L has above. For
example, it directly implies that L lies on the circumcircles of triangles IA IB IC and BCIA ,
and that the point L lies on SIA (since S = BC ∩ IB IC ). For this reason, many students
found it easier to think about the problem in terms of 4IA IB IC rather than 4ABC.
8
USAMO 2017 Solution Notes web.evanchen.cc, updated 2 June 2023
§2 Solutions to Day 2
§2.1 USAMO 2017/4, proposed by Maria Monks
Available online at https://aops.com/community/p8117190.
Problem statement
We present two solutions, one based on swapping and one based on an invariant.
¶ First “local” solution by swapping two points Let 1 ≤ i < n be any index and
consider the two red points Ri and Ri+1 . There are two blue points Bi and Bi+1 associated
with them.
Claim — If we swap the locations of points Ri and Ri+1 then the new arcs Ri → Bi
and Ri+1 → Bi+1 will cover the same points.
Proof. Delete all the points R1 , . . . , Ri−1 and B1 , . . . , Bi−1 ; instead focus on the positions
of Ri and Ri+1 .
The two blue points can then be located in three possible ways: either 0, 1, or 2 of
them lie on the arc Ri → Ri+1 . For each of the cases below, we illustrate on the left the
locations of Bi and Bi+1 and the corresponding arcs in green; then on the right we show
the modified picture where Ri and Ri+1 have swapped. (Note that by hypothesis there
are no other blue points in the green arcs).
9
USAMO 2017 Solution Notes web.evanchen.cc, updated 2 June 2023
Bi+1 Bi Bi+1 Bi
Bi Bi+1
Bi+1 Bi
Bi Bi+1 Bi Bi+1
Observe that in all cases, the number of arcs covering any given point on the circumference
is not changed. Consequently, this proves the claim.
Finally, it is enough to recall that any permutation of the red points can be achieved
by swapping consecutive points (put another way: (i i + 1) generates the permutation
group Sn ). This solves the problem.
Remark. This proof does not work if one tries to swap Ri and Rj if |i − j| 6= 1. For example
if we swapped Ri and Ri+2 then there are some issues caused by the possible presence of
the blue point Bi+1 in the green arc Ri+2 → Bi+2 .
¶ Second longer solution using an invariant Visually, if we draw all the segments
Ri → Bi then we obtain a set of n chords. Say a chord is inverted if satisfies the problem
condition, and stable otherwise. The problem contends that the number of stable/inverted
chords depends only on the layout of the points and not on the choice of chords.
−1 0
0 −1
(1, 0)
+1 0
0 −1
In fact we’ll describe the number of inverted chords explicitly. Starting from (1, 0) we
keep a running tally of R − B; in other words we start the counter at 0 and decrement
10
USAMO 2017 Solution Notes web.evanchen.cc, updated 2 June 2023
by 1 at each blue point and increment by 1 at each red point. Let x ≤ 0 be the lowest
number ever recorded. Then:
11
USAMO 2017 Solution Notes web.evanchen.cc, updated 2 June 2023
Problem statement
Find all real numbers c > 0 such that there exists a labeling of the lattice points in
Z2 with positive integers for which:
• for each label i, the distance between any two points labeled i is at least ci .
√
The answer is c < 2. Here is a solution
√ with Calvin Deng.
The construction for any c < 2 can be done as follows. Checkerboard color the
lattice points and label√the black ones with 1. The white points then form a copy of
Z2 again scaled up by 2 so we can repeat the procedure with 2 on half the resulting
points. Continue this dyadic construction until a large N for which cN < 2 2 (N −1) , at
1
A
2 1 2 1
1 5 1 3 6
2 1 2 1
1 3 1 4
Lemma
In a unit square, among any four points, two of these points have distance ≤ 1 apart.
Proof. Look at the four rays emanating from the origin and note that two of them have
included angle ≤ 90◦ .
12
USAMO 2017 Solution Notes web.evanchen.cc, updated 2 June 2023
So WLOG the northwest quadrant has no 2n’s. Take a 2n − 1 in the northwest and
draw a square of size 2n−1 × 2n−1 directly right of it (with its top edge coinciding with
the top of S). Then A can’t contain 2n − 1, so it must contain a 2n label; that 2n label
must be in the northeast quadrant.
Then we define a square B of size 2n−1 × 2n−1 as follows. If 2n − 1 is at least as high
2n, let B be a 2n−1 × 2n−1 square which touches 2n − 1 north and is bounded east by
2n. Otherwise let B be the square that touches 2n − 1 west and is bounded north by 2n.
We then observe B can neither have 2n − 1 nor 2n in it, contradiction.
13
USAMO 2017 Solution Notes web.evanchen.cc, updated 2 June 2023
Problem statement
Thus
X a a + b + c + d ab + bc + cd + da 1 2
≥ − ≥1− = .
cyc
b3 +4 4 12 3 3
Remark. The main interesting bit is the equality at (a, b, c, d) = (2, 2, 0, 0). P
This is the
main motivation for trying tangent line trick, since a lower bound of the form a(1 − λb)
preserves the unusual equality case above. Thus one takes the tangent at b = 2 which
miraculously passes through the point (0, 1/4) as well.
14